Mathcenter Forum

Mathcenter Forum (https://www.mathcenter.net/forum/index.php)
-   ปัญหาคณิตศาสตร์ ม. ต้น (https://www.mathcenter.net/forum/forumdisplay.php?f=31)
-   -   ข้อสอบ N.T ม.3 (https://www.mathcenter.net/forum/showthread.php?t=3708)

Rabbit 3/13 28 ธันวาคม 2007 21:52

ข้อสอบ N.T ม.3
 
เป็นเรื่องของการแปรผัน
โจทย์: ให้ A แปนผันโดยตรงกับกำลังสอง X และแปรผกผันกับรากที่สองที่เป็นบวกของ Y ถ้า X และ Y มีค่าเป็น 2 เท่าของค่าเดินแล้ว A จะมีค่าเป็นกี่เท่าของค่าเดิม

ช่วยบอกวิธีคิดหน่อยนะคะ :confused: :kaka:

คณิตศาสตร์ 28 ธันวาคม 2007 22:17

A แปรผันโดยตรงกับกำลังสอง X $ A = kX^{2} $ :)
กับแปรผกผันกับรากที่สองที่เป็นบวกของ y นั่นคือ A = $kX^{2}\frac{ k}{\sqrt{Y}}$ :kiki:

A = $\frac{k^{2}X^{2}}{\sqrt{Y}}$ ........(1) :sweat::sung:
นำสองมาคูณกับ X และ Y แล้วจะได้สมการที่ (2)
แล้วนำ (2)/(1) จะได้เป็น ...... เท่าจากค่าเดิม ครับ :great:

king_killer 29 ธันวาคม 2007 10:38

สมการที่ 1 ทำไมเป็น x กำลัง3 อ่ะ งับ บอกที

นายสบาย 29 ธันวาคม 2007 11:05

จะได้มีค่าเป็น $2\sqrt{2} $ของจำนวนเดิมนะครับ

king_killer 31 ธันวาคม 2007 08:01

ผมทำได้ 2 อ่ะครับ ทำไงได้ 2รูท2 อ่ะครับ บอกที ๆ :please:

TOP 31 ธันวาคม 2007 12:23

อ้างอิง:

ข้อความเดิมเขียนโดยคุณ คณิตศาสตร์ (ข้อความที่ 25886)
A แปรผันโดยตรงกับกำลังสอง X $ A = kX^{2} $ :)
กับแปรผกผันกับรากที่สองที่เป็นบวกของ y นั่นคือ A = $kX^{2}\frac{ k}{\sqrt{Y}}$ :kiki:

A = $\frac{k^{2}X^{2}}{\sqrt{Y}}$ ........(1) :sweat::sung:
นำสองมาคูณกับ X และ Y แล้วจะได้สมการที่ (2)
แล้วนำ (2)/(1) จะได้เป็น ...... เท่าจากค่าเดิม ครับ :great:

ค่าคงที่ $k$ ของการแปรผันทั้งสอง ไม่สามารถบอกได้ว่าเป็นค่าเดียวกันนะครับ เวลาเขียนแสดงวิธีทำต้องกำหนดเป็นคนละค่า เช่น $k_1$ กับ $k_2$ ดังนั้น
$A = k_1 X^2$ และ $A = \frac{k_2}{\sqrt{Y}}$
สรุปว่า $A = K_1 K_2 \frac{X^2}{\sqrt{Y}}$
และเราสามารถกำหนดค่าคงที่ตัวใหม่ $K_3 = K_1 K_2$ ได้เพื่อไม่ให้เทอมมันยุ่งจนเกินไป ก็จะได้
$A = K_3 \frac{X^2}{\sqrt{Y}}$

โจทย์บอกว่า $X , Y$ มีค่าเป็น 2 เท่า แทนค่าลงไป จะได้ $A_{ใหม่}$ เป็น
$A_{ใหม่} = K_3 \frac{(2X)^2}{\sqrt{2Y}}$
ดังนั้น $\frac{A_{ใหม่}}{A} = K_3 \frac{(2X)^2}{\sqrt{2Y}} \cdot \frac{1}{K_3} \frac{\sqrt{Y}}{X^2} = \frac{4}{\sqrt{2}} = 2\sqrt{2}$

หมายเหตุ: $k_1 = K_1 K_2 \frac{1}{\sqrt{Y}} $ และ $k_2 = K_1 K_2 X^2$

king_killer 31 ธันวาคม 2007 16:19

เข้าใจแล้วคับ ขอบคุณคุณ top มากครับ คือผมไปแทนค่า2เท่าของ y เป็น 2รูท y น่ะครับ เลยผิด :p

คณิตศาสตร์ 31 ธันวาคม 2007 18:44

ผมทำผิดเหรอครับเนี่ย

Rabbit 3/13 02 มกราคม 2008 21:26

ขอบคุณทุกคนมากเลยค่ะ

ขอบคุณทุกคนนะคะ ที่ให้วิธีคิดที่ดี

ขอบคุณทุกคนนะคะ ที่ให้วิธีคิดที่ดี เจ้าของกระทู้ไม่ได้เข้ามาดูเลย แต่ยังไงก็ขอบคุณค่ะ


เวลาที่แสดงทั้งหมด เป็นเวลาที่ประเทศไทย (GMT +7) ขณะนี้เป็นเวลา 17:06

Powered by vBulletin® Copyright ©2000 - 2024, Jelsoft Enterprises Ltd.
Modified by Jetsada Karnpracha